LSAT and Law School Admissions Forum

Get expert LSAT preparation and law school admissions advice from PowerScore Test Preparation.

User avatar
 Dave Killoran
PowerScore Staff
  • PowerScore Staff
  • Posts: 5852
  • Joined: Mar 25, 2011
|
#26432
Setup and Rule Diagram Explanation

This is a Pure Sequencing game.

Variables: J K L M N O P
October 1992_M12_L1_explanations_setup_diagram_1.png
* Either L or J must have received the most votes.
* Since N did not receive the fewest votes, it follows that M must have received the fewest votes.
You do not have the required permissions to view the files attached to this post.
 ginajoy14
  • Posts: 3
  • Joined: Apr 24, 2016
|
#23324
Hello-

In going through a pure sequencing logic game in the Lesson One PowerScore course book lesson section, I ran into some trouble understanding what some questions are asking. It is game number 1: October 1992 Questions 7-12. I am having trouble with questions 10-12... :-? Help!! :-D

Thank you!
User avatar
 Dave Killoran
PowerScore Staff
  • PowerScore Staff
  • Posts: 5852
  • Joined: Mar 25, 2011
|
#23325
Hi Gina,

Thanks for the questions! Let's briefly take a look at each stem:

Question #10: This question wants to know how many of the soft drinks are "fixed in place" based on the initial rules. So, if a soft drink is always last, for example, that would count as "one." If another soft drink was always third, that would add another, and so on. As it turns out, only M is fixed in place (only N or M could be last from the diagram, but the fourth rule then precludes N from being last, forcing M to be last), and so the correct answer is 1.

Question #11: There's a discussion of this question over at: viewtopic.php?f=431&t=11010 that you might find helpful. The basis of the question is to ask how many different soft drinks could be among the top 3. So, we need to look at the various solutions to the game. One solution to the game starts out as L-J-P, and so each of L, J, and P could be in the top 3, and we know at least 3 variables could be in it. Then, another solution starts out with L-P-N, and that adds N, a fourth variable to the mix. Are there any other solutions that start with any other variables besides L, J, P, and N? No, and so the correct answer is 4.

Question #12: This is the same question as #10, except a condition is added to the mix that P — J. Under that scenario, M is still last (as always) but L must be first, and P must be second. Thus, the correct answer is 3.


Please let me know if that helps!
 ginajoy14
  • Posts: 3
  • Joined: Apr 24, 2016
|
#23326
Yes, those explanations help. However, for number 12 can it be said/thought that it is asking, based on the given question, which ones have a permanency to their placement?
User avatar
 Dave Killoran
PowerScore Staff
  • PowerScore Staff
  • Posts: 5852
  • Joined: Mar 25, 2011
|
#23331
Yes, exactly! Both #10 and #12 are asking for the same thing. They just do it in different ways (and #12 has an extra condition) :-D
 Iceberg.Human
  • Posts: 4
  • Joined: Mar 04, 2020
|
#74176
Hi, I'm struggling to understand why this game's setup is like the above instead of something like the one I drew in the file. Comparing these two, I'm wondering when (in which circumstance) I should put LPOKM as the main diagram and when (in which circumstance) I should put JOKM diagram as the main diagram. (hope it makes sense!) Or whichever works the same? thank you!
You do not have the required permissions to view the files attached to this post.
User avatar
 KelseyWoods
PowerScore Staff
  • PowerScore Staff
  • Posts: 1079
  • Joined: Jun 26, 2013
|
#74190
Hi Iceberg!

Your setup is actually exactly the same as our setup! It may appear flipped, but you have all the same relationships. L is still before P. P is still before O and N. O is still before K, which is still before M. And J is still before O. Your setup is is a perfect representation of those rules. It does not actually matter whether you have N above or below the J-O-K-M chain. As long as the relationships are correct, the exact orientation of some of those branches does not matter.

The reason our setup looks like it does is because we actually started by using 2 separate chains before we combined everything together. So we combined the first 3 rules to get that J-O-K-M chain, and then we diagrammed the last rule (the LPNO rule) separately to make sure we didn't get too confused with all the relationships that we were keeping track of. When we branched N and O off of P, we had N above O, but it really makes no difference which one is above or below the other since there is no relationship between N and O. They are just 2 things that are after P. Once we saw that the 2 chains had O in common, we decided to add the straight-line J-O-K-M chain to the LPNO branched diagram just because it was a little easier to add the simpler diagram onto the more complicated one.

Hope this helps!

Best,
Kelsey
 Iceberg.Human
  • Posts: 4
  • Joined: Mar 04, 2020
|
#74205
Everything is clear! Thank you Kelsy!
 saygracealways
  • Posts: 34
  • Joined: Apr 09, 2020
|
#74861
Hi Powerscore,

Is there a reason why you chose to circle the variable N in this setup?

Thank you!
 Adam Tyson
PowerScore Staff
  • PowerScore Staff
  • Posts: 5153
  • Joined: Apr 14, 2011
|
#74880
That's just a way to indicate that there is that one special rule about N, that it cannot go last. Another way to show that would be a line drawn from N to M, indicating that N must be before M (since the rule about N forces M to be last, we can say "N is before M" and accomplish the same goal.)

Get the most out of your LSAT Prep Plus subscription.

Analyze and track your performance with our Testing and Analytics Package.